मोनोटोन अभिसरण प्रमेय: Difference between revisions

From Vigyanwiki
No edit summary
No edit summary
 
(6 intermediate revisions by 3 users not shown)
Line 1: Line 1:
{{short description|Theorems on the convergence of bounded monotonic sequences}}
{{short description|Theorems on the convergence of bounded monotonic sequences}}
[[वास्तविक विश्लेषण]] के गणितीय क्षेत्र में, '''मोनोटोन अभिसरण प्रमेय''' कई संबंधित प्रमेयों में से है जो [[मोनोटोनिक अनुक्रम]]ों (अनुक्रम जो गैर-घटते या गैर-बढ़ते हैं) के [[अभिसरण (गणित)]] को सिद्ध करना करते हैं जो कि [[बंधा हुआ कार्य]] भी हैं। अनौपचारिक रूप से, प्रमेय बताते हैं कि यदि कोई अनुक्रम बढ़ रहा है और ऊपर सर्वोच्च से घिरा हुआ है, तो अनुक्रम सर्वोच्च में परिवर्तित हो जाएगा; उसी तरह, यदि कोई अनुक्रम घट रहा है और नीचे अनंत से घिरा है, तो यह अनंत में परिवर्तित हो जाएगा।
[[वास्तविक विश्लेषण]] के गणितीय क्षेत्र में, '''मोनोटोन अभिसरण प्रमेय''' अनेक संबंधित प्रमेयों में से एक है जो [[मोनोटोनिक अनुक्रम|मोनोटोनिक अनुक्रमों]] (ऐसे अनुक्रम जो गैर-घटते या गैर-बढ़ते हैं) के [[अभिसरण (गणित)]] को सिद्ध करना करते हैं जो कि [[बंधा हुआ कार्य]] भी हैं। इस प्रकार अनौपचारिक रूप से, प्रमेय बताते हैं कि यदि कोई अनुक्रम बढ़ रहा है और ऊपर सर्वोच्च से घिरा हुआ है, तब अनुक्रम सर्वोच्च में परिवर्तित हो जाएगा; उसी तरह, यदि कोई अनुक्रम घट रहा है और नीचे एक अनंत से घिरा है, तब यह अनंत में परिवर्तित हो जाएगा।


==वास्तविक संख्याओं के मोनोटोन अनुक्रम का अभिसरण==
=='''वास्तविक संख्याओं के मोनोटोन अनुक्रम का अभिसरण'''==


===लेम्मा 1===
===लेम्मा 1===
यदि वास्तविक संख्याओं का अनुक्रम बढ़ रहा है और ऊपर से घिरा हुआ है, तो इसकी सर्वोच्च सीमा है।
यदि वास्तविक संख्याओं का अनुक्रम बढ़ रहा है और ऊपर से घिरा हुआ है, तब इसकी सर्वोच्च सीमा है।


===प्रमाण===
===प्रमाण===
होने देना <math> (a_n)_{n\in\mathbb{N}} </math> ऐसा क्रम हो, और चलो <math>\{ a_n \}</math> की शर्तों का सेट हो <math> (a_n)_{n\in\mathbb{N}} </math>. अनुमान से, <math>\{ a_n \}</math> गैर-रिक्त है और ऊपर से घिरा हुआ है। वास्तविक संख्याओं की [[न्यूनतम-ऊपरी-सीमा वाली संपत्ति]] द्वारा, <math display="inline">c = \sup_n \{a_n\}</math> अस्तित्व में है और सीमित है। अब, प्रत्येक के लिए <math>\varepsilon > 0</math>, वहां उपस्तिथ <math>N</math> ऐसा है कि <math>a_N > c - \varepsilon </math>, अन्यथा से <math>c - \varepsilon </math> की ऊपरी सीमा है <math>\{ a_n \}</math>, जो की परिभाषा के विपरीत है <math>c</math>. तब से <math>(a_n)_{n\in\mathbb{N}}</math> बढ़ रहा है, और <math>c</math> प्रत्येक के लिए इसकी ऊपरी सीमा है <math>n > N</math>, अपने पास <math>|c - a_n| \leq |c - a_N| < \varepsilon </math>. इसलिए, परिभाषा के अनुसार, की सीमा <math>(a_n)_{n\in\mathbb{N}}</math> है <math display="inline">\sup_n \{a_n\}.</math>
होने देना <math> (a_n)_{n\in\mathbb{N}} </math> ऐसा क्रम हो, और चलो <math>\{ a_n \}</math> की शर्तों का समुच्चय हो <math> (a_n)_{n\in\mathbb{N}} </math>. अनुमान से, <math>\{ a_n \}</math> गैर-रिक्त है और ऊपर से घिरा हुआ है। इस प्रकार वास्तविक संख्याओं की [[न्यूनतम-ऊपरी-सीमा वाली संपत्ति]] द्वारा, <math display="inline">c = \sup_n \{a_n\}</math> अस्तित्व में है और सीमित है। अभी, प्रत्येक के लिए <math>\varepsilon > 0</math>, वहां उपस्तिथ <math>N</math> ऐसा है कि <math>a_N > c - \varepsilon </math>, अन्यथा से <math>c - \varepsilon </math> की ऊपरी सीमा है <math>\{ a_n \}</math>, जो की परिभाषा के विपरीत है <math>c</math>. तब से <math>(a_n)_{n\in\mathbb{N}}</math> बढ़ रहा है, और <math>c</math> प्रत्येक के लिए इसकी ऊपरी सीमा है <math>n > N</math>, अपने पास है <math>|c - a_n| \leq |c - a_N| < \varepsilon </math>. इसलिए, परिभाषा के अनुसार, की सीमा <math>(a_n)_{n\in\mathbb{N}}</math> <math display="inline">\sup_n \{a_n\}.</math> है।
===लेम्मा 2===
===लेम्मा 2===
यदि वास्तविक संख्याओं का कोई क्रम घट रहा हो और नीचे परिबद्ध हो, तो उसकी न्यूनतम सीमा होती है।
यदि वास्तविक संख्याओं का कोई क्रम घट रहा हो और नीचे परिबद्ध हो, तब उसकी न्यूनतम सीमा होती है।


===प्रमाण===
===प्रमाण===
Line 16: Line 16:


===प्रमेय===
===प्रमेय===
अगर <math>(a_n)_{n\in\mathbb{N}}</math> [[वास्तविक संख्या]]ओं का मोनोटोन [[अनुक्रम]] है (अर्थात्, यदि a<sub>''n''</sub>≤ए<sub>''n''+1</sub> प्रत्येक n ≥ 1 या a के लिए<sub>''n''</sub>≥ए<sub>''n''+1</sub> प्रत्येक n ≥ 1) के लिए, तो इस अनुक्रम की सीमित सीमा होती है यदि और केवल यदि अनुक्रम परिबद्ध अनुक्रम है।<ref>A generalisation of this theorem was given by {{cite journal |first=John |last=Bibby |year=1974 |title=Axiomatisations of the average and a further generalisation of monotonic sequences |journal=[[Glasgow Mathematical Journal]] |volume=15 |issue=1 |pages=63–65 |doi=10.1017/S0017089500002135 |doi-access=free }}</ref>
यदि <math>(a_n)_{n\in\mathbb{N}}</math> [[वास्तविक संख्या]]ओं का मोनोटोन [[अनुक्रम]] है (अर्थात्, यदि ''a<sub>n</sub>'' ''a<sub>n</sub>''<sub>+1</sub> प्रत्येक ''n'' ≥ 1 और ''a<sub>n</sub>'' ''a<sub>n</sub>''<sub>+1</sub> प्रत्येक ''n'' ≥ 1) तो इस अनुक्रम की एक सीमित सीमा होती है यदि और केवल यदि अनुक्रम परिबद्ध अनुक्रम है।<ref>A generalisation of this theorem was given by {{cite journal |first=John |last=Bibby |year=1974 |title=Axiomatisations of the average and a further generalisation of monotonic sequences |journal=[[Glasgow Mathematical Journal]] |volume=15 |issue=1 |pages=63–65 |doi=10.1017/S0017089500002135 |doi-access=free }}</ref>
===प्रमाण===
===प्रमाण===
* यदि -दिशा: प्रमाण सीधे लेम्मास से आता है।
* '''"यदि"''' -दिशा: प्रमाण सीधे लेम्मा से आता है।
* केवल यदि -दिशा: (ε, δ) द्वारा - सीमा की परिभाषा, प्रत्येक अनुक्रम <math>(a_n)_{n\in\mathbb{N}}</math> सीमित सीमा के साथ <math>L</math> आवश्यक रूप से परिबद्ध है।
* '''"केवल यदि"''' -दिशा: (ε, δ) द्वारा - सीमा की परिभाषा, प्रत्येक अनुक्रम <math>(a_n)_{n\in\mathbb{N}}</math> सीमित सीमा के साथ <math>L</math> आवश्यक रूप से परिबद्ध है।


==एक मोनोटोन श्रृंखला का अभिसरण==
=='''एक मोनोटोन श्रृंखला का अभिसरण'''==


===प्रमेय===
===प्रमेय===
यदि सभी प्राकृत संख्याओं j और k के लिए, a<sub>''j'',''k''</sub> गैर-नकारात्मक वास्तविक संख्या है और a<sub>''j'',''k''</sub>≤ <sub>''j''+1,''k''</sub>, तब<ref>See for instance {{cite book |first=J. |last=Yeh |title=Real Analysis: Theory of Measure and Integration |location=Hackensack, NJ |publisher=World Scientific |year=2006 |isbn=981-256-653-8 }}</ref>{{rp|168}}
यदि सभी प्राकृतिक संख्याओं j और k के लिए, ''k'', ''a<sub>j</sub>''<sub>,''k''</sub> गैर-ऋणात्मक वास्तविक संख्या है और ''a<sub>j</sub>''<sub>,''k''</sub> ≤ ''a<sub>j</sub>''<sub>+1,''k''</sub>, तो<ref>See for instance {{cite book |first=J. |last=Yeh |title=Real Analysis: Theory of Measure and Integration |location=Hackensack, NJ |publisher=World Scientific |year=2006 |isbn=981-256-653-8 }}</ref>{{rp|168}}
:<math>\lim_{j\to\infty} \sum_k a_{j,k} = \sum_k \lim_{j\to\infty} a_{j,k}.</math>
:<math>\lim_{j\to\infty} \sum_k a_{j,k} = \sum_k \lim_{j\to\infty} a_{j,k}.</math>
प्रमेय कहता है कि यदि आपके पास गैर-नकारात्मक वास्तविक संख्याओं का अनंत मैट्रिक्स है
प्रमेय कहता है कि यदि आपके पास गैर-ऋणात्मक वास्तविक संख्याओं का अनंत आव्युह है
#कॉलम कमजोर रूप से बढ़ रहे हैं और बंधे हुए हैं, और
#कॉलम अशक्त रूप से बढ़ रहे हैं और बंधे हुए हैं, और
#प्रत्येक पंक्ति के लिए, [[श्रृंखला (गणित)]] जिसके पद इस पंक्ति द्वारा दिए गए हैं, का अभिसरण योग है,
#प्रत्येक पंक्ति के लिए, [[श्रृंखला (गणित)]] जिसके पद इस पंक्ति द्वारा दिए गए हैं, उसका एक अभिसरण योग है,
तो पंक्तियों के योग की सीमा उस श्रृंखला के योग के बराबर होती है जिसका पद k स्तंभ k की सीमा द्वारा दिया जाता है (जो इसका सर्वोच्च भी है)। श्रृंखला में अभिसरण योग होता है यदि और केवल यदि पंक्ति योगों का (कमजोर रूप से बढ़ता हुआ) क्रम परिबद्ध है और इसलिए अभिसरण है।
तब पंक्तियों के योग की सीमा उस श्रृंखला के योग के सामान्तर होती है जिसका पद k स्तंभ k की सीमा द्वारा दिया जाता है (जो इसका सर्वोच्च भी है)। श्रृंखला में अभिसरण योग होता है यदि और केवल यदि पंक्ति योगों का (अशक्त रूप से बढ़ता हुआ) क्रम परिबद्ध है और इसलिए अभिसरण है।


उदाहरण के तौर पर, पंक्तियों की अनंत श्रृंखला पर विचार करें
उदाहरण के तौर पर, पंक्तियों की अनंत श्रृंखला पर विचार करें


::<math> \left( 1+ \frac1 n\right)^n = \sum_{k=0}^n \binom nk \frac 1 {n^k} = \sum_{k=0}^n \frac1{k!} \times \frac nn \times \frac{n-1}n\times\cdots\times\frac{n-k+1}n,</math>
::<math> \left( 1+ \frac1 n\right)^n = \sum_{k=0}^n \binom nk \frac 1 {n^k} = \sum_{k=0}^n \frac1{k!} \times \frac nn \times \frac{n-1}n\times\cdots\times\frac{n-k+1}n,</math>
जहां n अनंत तक पहुंचता है (इस श्रृंखला की सीमा e (गणितीय स्थिरांक) है)। यहां पंक्ति n और कॉलम k में मैट्रिक्स प्रविष्टि है
जहां n अनंत तक पहुंचता है (इस श्रृंखला की सीमा e (गणितीय स्थिरांक) है)। यहां पंक्ति n और कॉलम k में आव्युह प्रविष्टि है


:<math>\binom nk \frac 1 {n^k} =\frac1{k!}\times\frac nn\times\frac{n-1}n\times\cdots\times\frac{n-k+1}n;</math>
:<math>\binom nk \frac 1 {n^k} =\frac1{k!}\times\frac nn\times\frac{n-1}n\times\cdots\times\frac{n-k+1}n;</math>
कॉलम (निश्चित k) वास्तव में n के साथ कमजोर रूप से बढ़ रहे हैं और (1/k से!) बंधे हुए हैं, जबकि पंक्तियों में केवल सीमित रूप से कई गैर-शून्य पद हैं, इसलिए शर्त 2 संतुष्ट है; प्रमेय अब कहता है कि आप पंक्ति योग की सीमा की गणना कर सकते हैं <math>(1+1/n)^n</math> अर्थात्, स्तंभ सीमाओं का योग लेकर<math>\frac1{k!}</math>.
कॉलम (निश्चित k) वास्तव में n के साथ अशक्त रूप से बढ़ रहे हैं और (1/k से!) बंधे हुए हैं, जबकि पंक्तियों में केवल सीमित रूप से अनेक गैर-शून्य पद हैं, इसलिए शर्त 2 संतुष्ट है; प्रमेय अभी कहता है कि आप पंक्ति योग की सीमा की गणना कर सकते हैं <math>(1+1/n)^n</math> अर्थात्, स्तंभ सीमाओं का योग लेकर<math>\frac1{k!}</math>.


=='''[[बेप्पो लेवी]] की लेम्मा'''==
=='''[[बेप्पो लेवी]] की लेम्मा'''==
निम्नलिखित परिणाम बेप्पो लेवी के कारण है, जिन्होंने 1906 में [[हेनरी लेबेस्गुए]] द्वारा पहले के परिणाम का थोड़ा सा सामान्यीकरण सिद्ध करना किया था।<ref>{{Citation
निम्नलिखित परिणाम बेप्पो लेवी के कारण है, जिन्होंने सत्र 1906 में [[हेनरी लेबेस्गुए|हेनरी लेब्सग्यू]] द्वारा पहले के परिणाम का थोड़ा सा सामान्यीकरण सिद्ध करना किया था।<ref>{{Citation
   | last1 = Schappacher
   | last1 = Schappacher
   | first1 = Norbert
   | first1 = Norbert
Line 58: Line 58:
   | page = 60
   | page = 60
| s2cid = 125072148
| s2cid = 125072148
  }}</ref> जो आगे हुआ, <math>\operatorname{\mathcal B}_{\R_{\geq 0}}</math> को दर्शाता है <math>\sigma</math>- बोरेल का बीजगणित चालू होता है <math>[0,+\infty]</math>. परिभाषा से, <math>\operatorname{\mathcal B}_{\R_{\geq 0}}</math> सेट सम्मिलित है <math>\{+\infty\}</math> और सभी बोरेल उपसमुच्चय <math>\R_{\geq 0}.</math>
  }}</ref> इस प्रकार जो आगे हुआ, <math>\operatorname{\mathcal B}_{\R_{\geq 0}}</math> को दर्शाता है <math>\sigma</math>- बोरेल का बीजगणित चालू होता है <math>[0,+\infty]</math>. परिभाषा से, <math>\operatorname{\mathcal B}_{\R_{\geq 0}}</math> समुच्चय सम्मिलित है <math>\{+\infty\}</math> और सभी बोरेल उपसमुच्चय <math>\R_{\geq 0}.</math>
===प्रमेय===
===प्रमेय===
होने देना <math>(\Omega,\Sigma,\mu)</math> माप हो (गणित), और <math>X\in\Sigma</math>. बिंदुवार गैर-घटते क्रम पर विचार करें <math>\{f_k\}^\infty_{k=1}</math> का <math>(\Sigma,\operatorname{\mathcal B}_{\R_{\geq 0}})</math>-[[मापने योग्य कार्य]] गैर-नकारात्मक कार्य <math>f_k:X\to [0,+\infty]</math>, अर्थात, प्रत्येक के लिए <math>{k\geq 1}</math> और हर <math>{x\in X}</math>,
होने देना <math>(\Omega,\Sigma,\mu)</math> माप हो (गणित), और <math>X\in\Sigma</math>. बिंदुवार गैर-घटते क्रम पर विचार करें <math>\{f_k\}^\infty_{k=1}</math> का <math>(\Sigma,\operatorname{\mathcal B}_{\R_{\geq 0}})</math>-[[मापने योग्य कार्य]] गैर-ऋणात्मक कार्य <math>f_k:X\to [0,+\infty]</math>, अर्थात, प्रत्येक के लिए <math>{k\geq 1}</math> और हर <math>{x\in X}</math>,


:<math> 0 \leq f_k(x) \leq f_{k+1}(x)\leq\infty. </math>
:<math> 0 \leq f_k(x) \leq f_{k+1}(x)\leq\infty. </math>
Line 69: Line 69:


:<math>\lim_{k\to\infty} \int_X f_k \,d\mu = \int_X f \,d\mu. </math>
:<math>\lim_{k\to\infty} \int_X f_k \,d\mu = \int_X f \,d\mu. </math>
टिप्पणी 1. अभिन्न अंग परिमित या अनंत हो सकते हैं।
'''टिप्पणी 1.''' अभिन्न अंग परिमित या अनंत हो सकते हैं।


टिप्पणी 2. यदि इसकी धारणाएँ मान्य हैं तो प्रमेय सत्य रहता है <math>\mu</math>-लगभग हर जगह। दूसरे शब्दों में, यह पर्याप्त है कि [[शून्य सेट]] है <math>N</math> ऐसा कि क्रम <math>\{f_n(x)\}</math> प्रत्येक के लिए गैर-कमी <math>{x\in X\setminus N}.</math> यह देखने के लिए कि यह सच क्यों है, हम अवलोकन से प्रारंभ करते हैं जो अनुक्रम की अनुमति देता है <math>\{ f_n \}</math> बिंदुवार गैर-घटाना लगभग हर जगह इसकी बिंदुवार सीमा का कारण बनता है <math>f</math> कुछ शून्य सेट पर अपरिभाषित होना <math>N</math>. उस शून्य सेट पर, <math>f</math> फिर इच्छानुसार से परिभाषित किया जा सकता है, उदाहरण के लिए शून्य के रूप में, या किसी अन्य तरीके से जो मापनीयता को सुरक्षित रखता है। यह देखने के लिए कि यह प्रमेय के परिणाम को प्रभावित क्यों नहीं करेगा, तब से ध्यान दें <math>{\mu(N)=0},</math> हमारे पास, हर किसी के लिए है <math>k,</math>
'''टिप्पणी 2.''' यदि इसकी धारणाएँ मान्य हैं तब प्रमेय सत्य रहता है <math>\mu</math>-लगभग हर स्थान। दूसरे शब्दों में, यह पर्याप्त है कि [[शून्य सेट|शून्य समुच्चय]] है <math>N</math> ऐसा कि क्रम <math>\{f_n(x)\}</math> प्रत्येक के लिए गैर-कमी <math>{x\in X\setminus N}.</math> यह देखने के लिए कि यह सच क्यों है, हम अवलोकन से प्रारंभ करते हैं जो अनुक्रम की अनुमति देता है <math>\{ f_n \}</math> बिंदुवार गैर-घटाना लगभग हर स्थान इसकी बिंदुवार सीमा का कारण बनता है <math>f</math> कुछ शून्य समुच्चय पर अपरिभाषित होना <math>N</math>. उस शून्य समुच्चय पर, <math>f</math> फिर इच्छानुसार से परिभाषित किया जा सकता है, उदाहरण के लिए शून्य के रूप में, या किसी अन्य तरीके से जो मापनीयता को सुरक्षित रखता है। यह देखने के लिए कि यह प्रमेय के परिणाम को प्रभावित क्यों नहीं करेगा, तब से ध्यान दें <math>{\mu(N)=0},</math> हमारे पास, हर किसी के लिए है <math>k,</math>
:<math> \int_X f_k \,d\mu = \int_{X \setminus N} f_k \,d\mu</math> और <math>\int_X f \,d\mu = \int_{X \setminus N} f \,d\mu, </math>
:<math> \int_X f_k \,d\mu = \int_{X \setminus N} f_k \,d\mu</math> और <math>\int_X f \,d\mu = \int_{X \setminus N} f \,d\mu, </math>
उसे उपलब्ध कराया <math>f</math> है <math>(\Sigma,\operatorname{\mathcal B}_{\R_{\geq 0}})</math>-मापने योग्य.<ref name="SCHECHTER1997">See for instance {{cite book |first=Erik |last=Schechter |title=Handbook of Analysis and Its Foundations |location=San Diego |publisher=Academic Press |year=1997 |isbn=0-12-622760-8 }}</ref>{{rp|at=section 21.38}} (ये समानताएं गैर-नकारात्मक फ़ंक्शन के लिए लेबेस्ग इंटीग्रल की परिभाषा से सीधे अनुसरण करती हैं)।
उसे उपलब्ध कराया <math>f</math> है <math>(\Sigma,\operatorname{\mathcal B}_{\R_{\geq 0}})</math>-मापने योग्य.<ref name="SCHECHTER1997">See for instance {{cite book |first=Erik |last=Schechter |title=Handbook of Analysis and Its Foundations |location=San Diego |publisher=Academic Press |year=1997 |isbn=0-12-622760-8 }}</ref>{{rp|at=धारा 21.38}} (यह समानताएं गैर-ऋणात्मक फलन के लिए लेबेस्ग इंटीग्रल की परिभाषा से सीधे अनुसरण करती हैं)।


टिप्पणी 3. प्रमेय की मान्यताओं के अनुसार ,
'''टिप्पणी 3.''' प्रमेय की मान्यताओं के अनुसार ,
{{ordered list|type=lower-alpha
{{ordered list|type=lower-alpha
| <math>\textstyle f(x) = \liminf_k f_k(x) = \limsup_k f_k(x) = \sup_k f_k(x)</math>
| <math>\textstyle f(x) = \liminf_k f_k(x) = \limsup_k f_k(x) = \sup_k f_k(x)</math>
Line 83: Line 83:
(ध्यान दें कि समानता की दूसरी श्रृंखला टिप्पणी 5 से अनुसरण करती है)।
(ध्यान दें कि समानता की दूसरी श्रृंखला टिप्पणी 5 से अनुसरण करती है)।


टिप्पणी 4. नीचे दिया गया प्रमाण यहां स्थापित किए गए को छोड़कर लेबेस्ग इंटीग्रल के किसी भी गुण का उपयोग नहीं करता है। इस प्रकार, प्रमेय का उपयोग लेबेस्ग एकीकरण से संबंधित अन्य बुनियादी गुणों, जैसे कि रैखिकता, को सिद्ध करना करने के लिए किया जा सकता है।
'''टिप्पणी 4.''' नीचे दिया गया प्रमाण यहां स्थापित किए गए को छोड़कर लेबेस्ग इंटीग्रल के किसी भी गुण का उपयोग नहीं करता है। इस प्रकार, प्रमेय का उपयोग लेबेस्ग एकीकरण से संबंधित अन्य बुनियादी गुणों, जैसे कि रैखिकता, को सिद्ध करना करने के लिए किया जा सकता है।


टिप्पणी 5 (लेबेस्ग इंटीग्रल की एकरसता)। नीचे दिए गए प्रमाण में, हम लेबेस्ग इंटीग्रल के मोनोटोनिक गुण को केवल गैर-नकारात्मक कार्यों पर प्रयुक्त करते हैं। विशेष रूप से (टिप्पणी 4 देखें), कार्य करें <math>f,g : X \to [0,+\infty]</math> होना <math>(\Sigma,\operatorname{\mathcal B}_{\R_{\geq 0}})</math>-मापने योग्य.
'''टिप्पणी 5''' (लेबेस्ग इंटीग्रल की एकरसता)। नीचे दिए गए प्रमाण में, हम लेबेस्ग इंटीग्रल के मोनोटोनिक गुण को केवल गैर-ऋणात्मक कार्यों पर प्रयुक्त करते हैं। विशेष रूप से (टिप्पणी 4 देखें), कार्य करें <math>f,g : X \to [0,+\infty]</math> होना <math>(\Sigma,\operatorname{\mathcal B}_{\R_{\geq 0}})</math>-मापने योग्य.


*अगर <math>f \leq g</math> हर जगह पर <math>X,</math> तब
*यदि <math>f \leq g</math> हर स्थान पर <math>X,</math> तब


:<math>\int_X f\,d\mu \leq \int_X g\,d\mu.</math>
:<math>\int_X f\,d\mu \leq \int_X g\,d\mu.</math>
*अगर <math> X_1,X_2 \in \Sigma </math> और <math>X_1 \subseteq X_2, </math> तब
*यदि <math> X_1,X_2 \in \Sigma </math> और <math>X_1 \subseteq X_2, </math> तब


:<math>\int_{X_1} f\,d\mu \leq \int_{X_2} f\,d\mu.</math>
:<math>\int_{X_1} f\,d\mu \leq \int_{X_2} f\,d\mu.</math>
सबूत। निरूपित <math>\operatorname{SF}(h)</math> सरल का सेट <math>(\Sigma, \operatorname{\mathcal B}_{\R_{\geq 0}})</math>-मापने योग्य कार्य <math>s:X\to [0,\infty)</math> ऐसा है कि
प्रमाण। निरूपित <math>\operatorname{SF}(h)</math> सरल का समुच्चय <math>(\Sigma, \operatorname{\mathcal B}_{\R_{\geq 0}})</math>-मापने योग्य कार्य <math>s:X\to [0,\infty)</math> ऐसा है कि
<math>0\leq s\leq h</math> हर जगह पर <math>X.</math> 1. चूँकि <math>f \leq g,</math> अपने पास
<math>0\leq s\leq h</math> हर स्थान पर <math>X.</math> 1. चूँकि <math>f \leq g,</math> अपने पास


:<math> \operatorname{SF}(f) \subseteq \operatorname{SF}(g). </math>
:<math> \operatorname{SF}(f) \subseteq \operatorname{SF}(g). </math>
Line 100: Line 100:


:<math>\int_X f\,d\mu = \sup_{s\in {\rm SF}(f)}\int_X s\,d\mu \leq \sup_{s\in {\rm SF}(g)}\int_X s\,d\mu = \int_X g\,d\mu.</math>
:<math>\int_X f\,d\mu = \sup_{s\in {\rm SF}(f)}\int_X s\,d\mu \leq \sup_{s\in {\rm SF}(g)}\int_X s\,d\mu = \int_X g\,d\mu.</math>
2. चलो <math>{\mathbf 1}_{X_1}</math> सेट का सूचक कार्य हो <math>X_1.</math> इसका अनुमान लेबेस्ग इंटीग्रल की परिभाषा से लगाया जा सकता है
2. चलो <math>{\mathbf 1}_{X_1}</math> समुच्चय का सूचक कार्य हो <math>X_1.</math> इसका अनुमान लेबेस्ग इंटीग्रल की परिभाषा से लगाया जा सकता है


:<math> \int_{X_2} f\cdot {\mathbf 1}_{X_1} \,d\mu = \int_{X_1} f \,d\mu</math>
:<math> \int_{X_2} f\cdot {\mathbf 1}_{X_1} \,d\mu = \int_{X_1} f \,d\mu</math>
Line 107: Line 107:
:<math> \int_{X_1} f \,d\mu = \int_{X_2} f\cdot {\mathbf 1}_{X_1} \,d\mu \leq \int_{X_2} f \,d\mu. </math>
:<math> \int_{X_1} f \,d\mu = \int_{X_2} f\cdot {\mathbf 1}_{X_1} \,d\mu \leq \int_{X_2} f \,d\mu. </math>
===प्रमाण===
===प्रमाण===
यह प्रमाण फ़तौ की प्रमेयिका पर निर्भर नहीं करता है; चूँकि, हम बताते हैं कि उस लेम्मा का उपयोग कैसे किया जा सकता है। जो लोग प्रमाण की इस स्वतंत्रता में रुचि नहीं रखते हैं वे नीचे दिए गए मध्यवर्ती परिणामों को छोड़ सकते हैं।
यह प्रमाण फ़तौ की प्रमेयिका पर निर्भर नहीं करता है; चूँकि, हम बताते हैं कि उस लेम्मा का उपयोग कैसे किया जा सकता है। जो लोग प्रमाण की इस स्वतंत्रता में रुचि नहीं रखते हैं वह नीचे दिए गए मध्यवर्ती परिणामों को छोड़ सकते हैं।


====मध्यवर्ती परिणाम====
====मध्यवर्ती परिणाम====


=====लेब्सग्यू माप के रूप में अभिन्न =====
=====लेब्सग्यू माप के रूप में अभिन्न =====
लेम्मा 1. चलो <math>(\Omega,\Sigma,\mu)</math> मापने योग्य स्थान बनें. सरल पर विचार करें <math>(\Sigma,\operatorname{\mathcal B}_{\R_{\geq 0}})</math>-मापने योग्य गैर-नकारात्मक कार्य <math>s:\Omega\to{\mathbb R_{\geq 0}}</math>. उपसमुच्चय के लिए <math>S\subseteq\Omega</math>, परिभाषित करना
लेम्मा 1. चलो <math>(\Omega,\Sigma,\mu)</math> मापने योग्य स्थान बनें. सरल पर विचार करें <math>(\Sigma,\operatorname{\mathcal B}_{\R_{\geq 0}})</math>-मापने योग्य गैर-ऋणात्मक कार्य <math>s:\Omega\to{\mathbb R_{\geq 0}}</math>. उपसमुच्चय के लिए <math>S\subseteq\Omega</math>, परिभाषित करना
:<math>\nu(S)=\int_Ss\,d\mu.</math>
:<math>\nu(S)=\int_Ss\,d\mu.</math>
तब <math>\nu</math> पर उपाय है <math>\Omega</math>.
तब <math>\nu</math> पर उपाय है <math>\Omega</math>.


======प्रमाण======
======प्रमाण======
एकरसता टिप्पणी 5 से आती है। यहां, हम केवल गणनीय योगात्मकता सिद्ध करेंगे, बाकी पाठक पर छोड़ देंगे। होने देना <math>S=\bigcup^\infty_{i=1}S_i</math>, जहां सभी सेट <math>S_i</math> जोड़ीवार असंयुक्त हैं. सरलता के कारण,
एकरसता टिप्पणी 5 से आती है। यहां, हम केवल गणनीय योगात्मकता सिद्ध करेंगे, बाकी पाठक पर छोड़ देंगे। होने देना <math>S=\bigcup^\infty_{i=1}S_i</math>, जहां सभी समुच्चय <math>S_i</math> जोड़ीवार असंयुक्त हैं. सरलता के कारण,
:<math>s=\sum^n_{i=1}c_i\cdot {\mathbf 1}_{A_i},</math>
:<math>s=\sum^n_{i=1}c_i\cdot {\mathbf 1}_{A_i},</math>
कुछ परिमित गैर-ऋणात्मक स्थिरांकों के लिए <math>c_i\in{\mathbb R}_{\geq 0}</math> और जोड़ीवार असंयुक्त सेट <math>A_i\in\Sigma</math> ऐसा है कि <math>\bigcup^n_{i=1}A_i=\Omega</math>. लेब्सेग इंटीग्रल की परिभाषा के अनुसार,
कुछ परिमित गैर-ऋणात्मक स्थिरांकों के लिए <math>c_i\in{\mathbb R}_{\geq 0}</math> और जोड़ीवार असंयुक्त समुच्चय <math>A_i\in\Sigma</math> ऐसा है कि <math>\bigcup^n_{i=1}A_i=\Omega</math>. लेब्सेग इंटीग्रल की परिभाषा के अनुसार,
:<math>
:<math>
\begin{align}
\begin{align}
Line 128: Line 128:
\end{align}
\end{align}
</math>
</math>
चूंकि सभी सेट <math>S_j\cap A_i</math> जोड़ीवार असंयुक्त हैं, गणनीय योगात्मकता <math>\mu</math>
चूंकि सभी समुच्चय <math>S_j\cap A_i</math> जोड़ीवार असंयुक्त हैं, गणनीय योगात्मकता <math>\mu</math>
 
हमें देता है
हमें देता है
:<math>
:<math>
\sum^n_{i=1} c_i\cdot\mu \left(\bigcup^\infty_{j=1}(S_j\cap A_i)\right)=\sum^n_{i=1}c_i\cdot\sum^\infty_{j=1} \mu(S_j\cap A_i).
\sum^n_{i=1} c_i\cdot\mu \left(\bigcup^\infty_{j=1}(S_j\cap A_i)\right)=\sum^n_{i=1}c_i\cdot\sum^\infty_{j=1} \mu(S_j\cap A_i).
</math>
</math>
चूँकि सभी सारांश गैर-ऋणात्मक हैं, श्रृंखला का योग, चाहे यह योग परिमित हो या अनंत, यदि योग क्रम बदलता है तो नहीं बदल सकता। इसी कारणवश,
चूँकि सभी सारांश गैर-ऋणात्मक हैं, श्रृंखला का योग, चाहे यह योग परिमित हो या अनंत, यदि योग क्रम बदलता है तब नहीं बदल सकता। इसी कारणवश,
:<math>
:<math>
\begin{align}
\begin{align}
Line 144: Line 145:
आवश्यकता अनुसार।
आवश्यकता अनुसार।


===== नीचे से निरंतरता =====
===== "नीचे से निरंतरता" =====
निम्नलिखित संपत्ति माप की परिभाषा का प्रत्यक्ष परिणाम है।
निम्नलिखित संपत्ति माप की परिभाषा का प्रत्यक्ष परिणाम है।


Line 151: Line 152:
S_1\subseteq\cdots\subseteq S_i\subseteq S_{i+1}\subseteq\cdots\subseteq S
S_1\subseteq\cdots\subseteq S_i\subseteq S_{i+1}\subseteq\cdots\subseteq S
</math>
</math>
अपने सभी सेटों के साथ गैर-घटती हुई श्रृंखला है <math>\mu</math>-मापने योग्य. तब
अपने सभी समुच्चयों के साथ गैर-घटती हुई श्रृंखला है <math>\mu</math>-मापने योग्य. तब
:<math>\mu(S)=\lim_i\mu(S_i).</math>
:<math>\mu(S)=\lim_i\mu(S_i).</math>
===='''प्रमेय का प्रमाण'''====
===='''प्रमेय का प्रमाण'''====
चरण 1. हम इसे दिखाकर शुरुआत करते हैं <math>f</math> है <math> (\Sigma, \operatorname{\mathcal B}_{\R_{\geq 0}}) </math>-मापने योग्य.<ref name="SCHECHTER1997"/>{{rp|at=section 21.3}}
'''चरण 1.''' हम इसे दिखाकर शुरुआत करते हैं <math>f</math> है <math> (\Sigma, \operatorname{\mathcal B}_{\R_{\geq 0}}) </math>-मापने योग्य.<ref name="SCHECHTER1997"/>{{rp|at=धारा 21.3}}


टिप्पणी। यदि हम फ़तौ की लेम्मा का उपयोग कर रहे थे, तो मापनीयता टिप्पणी 3(ए) से आसानी से अनुसरण करेगी।
टिप्पणी। यदि हम फ़तौ की लेम्मा का उपयोग कर रहे थे, तब मापनीयता टिप्पणी 3(ए) से आसानी से अनुसरण करेगी।


फ़तौ के लेम्मा का उपयोग किए बिना ऐसा करने के लिए, यह दिखाना पर्याप्त है कि अंतराल की उलटी छवि <math>[0,t]</math> अंतर्गत <math>f</math> [[सिग्मा-बीजगणित]] का तत्व है <math>\Sigma</math> पर <math>X</math>, क्योंकि (बंद) अंतराल वास्तविक पर [[बोरेल सिग्मा बीजगणित]] उत्पन्न करते हैं। तब से <math>[0,t]</math> बंद अंतराल है, और, प्रत्येक के लिए <math>k</math>, <math>0\le f_k(x) \le f(x)</math>,
फ़तौ के लेम्मा का उपयोग किए बिना ऐसा करने के लिए, यह दिखाना पर्याप्त है कि अंतराल की उलटी छवि <math>[0,t]</math> अंतर्गत <math>f</math> [[सिग्मा-बीजगणित]] का तत्व है <math>\Sigma</math> पर <math>X</math>, क्योंकि (बंद) अंतराल वास्तविक पर [[बोरेल सिग्मा बीजगणित|'''बोरेल सिग्मा बीजगणित''']] उत्पन्न करते हैं। तब से <math>[0,t]</math> बंद अंतराल है, और, प्रत्येक के लिए <math>k</math>, <math>0\le f_k(x) \le f(x)</math>,


:<math>0\leq f(x)\leq t\quad \Leftrightarrow\quad \Bigl[\forall k\quad 0\leq f_k(x)\leq t\Bigr].</math>
:<math>0\leq f(x)\leq t\quad \Leftrightarrow\quad \Bigl[\forall k\quad 0\leq f_k(x)\leq t\Bigr].</math>
Line 164: Line 165:


:<math>\{x\in X \mid 0\leq f(x)\leq t\} = \bigcap_k \{x\in X \mid 0\leq f_k(x)\leq t\}.</math>
:<math>\{x\in X \mid 0\leq f(x)\leq t\} = \bigcap_k \{x\in X \mid 0\leq f_k(x)\leq t\}.</math>
एक के नीचे स्थापित बोरेल की उलटी छवि होना <math>(\Sigma,\operatorname{\mathcal B}_{\R_{\geq 0}})</math>-मापने योग्य कार्य <math>f_k</math>, गणनीय प्रतिच्छेदन में प्रत्येक सेट का तत्व है <math>\Sigma</math>. तब से <math>\sigma</math>-बीजगणित, परिभाषा के अनुसार, गणनीय प्रतिच्छेदन के अंतर्गत बंद होते हैं, इससे पता चलता है <math>f</math> है <math>(\Sigma,\operatorname{\mathcal B}_{\R_{\geq 0}})</math>-मापने योग्य, और अभिन्न <math>\textstyle \int_X f \,d\mu </math> अच्छी तरह से परिभाषित है (और संभवतः अनंत)।
एक के नीचे स्थापित बोरेल की उलटी छवि होना <math>(\Sigma,\operatorname{\mathcal B}_{\R_{\geq 0}})</math>-मापने योग्य कार्य <math>f_k</math>, गणनीय प्रतिच्छेदन में प्रत्येक समुच्चय का तत्व है <math>\Sigma</math>. तब से <math>\sigma</math>-बीजगणित, परिभाषा के अनुसार, गणनीय प्रतिच्छेदन के अंतर्गत बंद होते हैं, इससे पता चलता है <math>f</math> है <math>(\Sigma,\operatorname{\mathcal B}_{\R_{\geq 0}})</math>-मापने योग्य, और अभिन्न <math>\textstyle \int_X f \,d\mu </math> अच्छी तरह से परिभाषित है (और संभवतः अनंत)।


स्टेप 2. हम सबसे पहले वो दिखाएंगे <math>\textstyle\int_X f \,d\mu \geq \lim_k \int_X f_k \,d\mu. </math>
'''चरण 2.''' हम सबसे पहले वो दिखाएंगे <math>\textstyle\int_X f \,d\mu \geq \lim_k \int_X f_k \,d\mu. </math>
की परिभाषा <math>f</math> और की एकरसता <math>\{f_k\}</math> इसका कारणयह है <math>f(x)\geq f_k(x)</math>, हरएक के लिए <math>k</math> और हर <math>x\in X</math>. लेबेस्ग इंटीग्रल की एकरसता (या, अधिक त्रुटिहीन रूप से, रिमार्क 5 में स्थापित इसका संकीर्ण संस्करण; रिमार्क 4 भी देखें) द्वारा,
की परिभाषा <math>f</math> और की एकरसता <math>\{f_k\}</math> इसका कारणयह है <math>f(x)\geq f_k(x)</math>, हरएक के लिए <math>k</math> और हर <math>x\in X</math>. लेबेस्ग इंटीग्रल की एकरसता (या, अधिक त्रुटिहीन रूप से, रिमार्क 5 में स्थापित इसका संकीर्ण संस्करण; रिमार्क 4 भी देखें) द्वारा,
:<math>\int_X f\,d\mu\geq\int_X f_k\,d\mu,</math>
:<math>\int_X f\,d\mu\geq\int_X f_k\,d\mu,</math>
Line 173: Line 174:
ध्यान दें कि दाईं ओर की सीमा उपस्तिथ है (सीमित या अनंत) क्योंकि, एकरसता के कारण (टिप्पणी 5 और टिप्पणी 4 देखें), अनुक्रम गैर-घटता नहीं है।
ध्यान दें कि दाईं ओर की सीमा उपस्तिथ है (सीमित या अनंत) क्योंकि, एकरसता के कारण (टिप्पणी 5 और टिप्पणी 4 देखें), अनुक्रम गैर-घटता नहीं है।


चरण 2 का अंत.
'''चरण 2''' का अंत.


अब हम विपरीत असमानता को सिद्ध करते हैं। हम यह दिखाना चाहते हैं
अभी हम विपरीत असमानता को सिद्ध करते हैं। हम यह दिखाना चाहते हैं


:<math> \int_X f \,d\mu \leq \lim_k \int_X f_k \,d\mu </math>.
:<math> \int_X f \,d\mu \leq \lim_k \int_X f_k \,d\mu </math>.
Line 181: Line 182:
फ़तौ की लेम्मा का उपयोग करके प्रमाण। टिप्पणी 3 के अनुसार, जिस असमानता को हम सिद्ध करना चाहते हैं वह इसके समतुल्य है
फ़तौ की लेम्मा का उपयोग करके प्रमाण। टिप्पणी 3 के अनुसार, जिस असमानता को हम सिद्ध करना चाहते हैं वह इसके समतुल्य है
:<math>\int_X \liminf_k f_k(x) \,d\mu \leq \liminf_k \int_X f_k \,d\mu.</math>
:<math>\int_X \liminf_k f_k(x) \,d\mu \leq \liminf_k \int_X f_k \,d\mu.</math>
किन्तु बाद वाला फ़तौ की लेम्मा से तुरंत अनुसरण करता है, और प्रमाण पूरा हो गया है।
किन्तु पश्चात् वाला फ़तौ की लेम्मा से तुरंत अनुसरण करता है, और प्रमाण पूरा हो गया है।


स्वतंत्र प्रमाण. फ़तौ की लेम्मा का उपयोग किए बिना असमानता को सिद्ध करना करने के लिए, हमें कुछ अतिरिक्त मशीनरी की आवश्यकता है। निरूपित <math>\operatorname{SF}(f)</math> सरल का सेट <math>(\Sigma,\operatorname{\mathcal B}_{\R_{\geq 0}})</math>-मापने योग्य कार्य <math>s:X\to [0,\infty)</math> ऐसा है कि
स्वतंत्र प्रमाण. फ़तौ की लेम्मा का उपयोग किए बिना असमानता को सिद्ध करना करने के लिए, हमें कुछ अतिरिक्त मशीनरी की आवश्यकता है। निरूपित <math>\operatorname{SF}(f)</math> सरल का समुच्चय <math>(\Sigma,\operatorname{\mathcal B}_{\R_{\geq 0}})</math>-मापने योग्य कार्य <math>s:X\to [0,\infty)</math> ऐसा है कि
<math>0\leq s\leq f</math> पर <math>X</math>.
<math>0\leq s\leq f</math> पर <math>X</math>.


चरण 3. सरल कार्य दिया गया है <math>s\in\operatorname{SF}(f)</math> और वास्तविक संख्या <math>t\in (0,1)</math>, परिभाषित करना
'''चरण 3.''' सरल कार्य दिया गया है <math>s\in\operatorname{SF}(f)</math> और वास्तविक संख्या <math>t\in (0,1)</math>, परिभाषित करना
:<math>B^{s,t}_k=\{x\in X\mid t\cdot s(x)\leq f_k(x)\}\subseteq X.</math>
:<math>B^{s,t}_k=\{x\in X\mid t\cdot s(x)\leq f_k(x)\}\subseteq X.</math>
तब <math>B^{s,t}_k\in\Sigma</math>, <math>B^{s,t}_k\subseteq B^{s,t}_{k+1}</math>, और <math>\textstyle X=\bigcup_k B^{s,t}_k</math>.
तब <math>B^{s,t}_k\in\Sigma</math>, <math>B^{s,t}_k\subseteq B^{s,t}_{k+1}</math>, और <math>\textstyle X=\bigcup_k B^{s,t}_k</math>.


चरण 3ए. पहले दावे को सिद्ध करना करने के लिए आइए <math>\textstyle s=\sum^m_{i=1}c_i\cdot{\mathbf 1}_{A_i}</math>, जोड़ीवार असंयुक्त मापन योग्य सेटों के कुछ सीमित संग्रह के लिए <math>A_i\in\Sigma</math> ऐसा है कि <math>\textstyle X=\cup^m_{i=1}A_i</math>, कुछ (परिमित) गैर-नकारात्मक स्थिरांक <math>c_i\in {\mathbb R}_{\geq 0}</math>, और <math>{\mathbf 1}_{A_i}</math> सेट के सूचक फ़ंक्शन को दर्शाते हुए <math>A_i</math>.
'''चरण 3ए.''' पहले दावे को सिद्ध करना करने के लिए आइए <math>\textstyle s=\sum^m_{i=1}c_i\cdot{\mathbf 1}_{A_i}</math>, जोड़ीवार असंयुक्त मापन योग्य समुच्चयों के कुछ सीमित संग्रह के लिए <math>A_i\in\Sigma</math> ऐसा है कि <math>\textstyle X=\cup^m_{i=1}A_i</math>, कुछ (परिमित) गैर-ऋणात्मक स्थिरांक <math>c_i\in {\mathbb R}_{\geq 0}</math>, और <math>{\mathbf 1}_{A_i}</math> समुच्चय के सूचक फलन को दर्शाते हुए <math>A_i</math>.


हरएक के लिए <math> x\in A_i, </math> <math>t\cdot s(x)\leq f_k(x)</math> यदि और केवल यदि धारण करता है <math> f_k(x) \in [t\cdot c_i, +\infty].</math> यह देखते हुए कि सेट <math>A_i</math> जोड़ीवार असंयुक्त हैं,
हरएक के लिए <math> x\in A_i, </math> <math>t\cdot s(x)\leq f_k(x)</math> यदि और केवल यदि धारण करता है <math> f_k(x) \in [t\cdot c_i, +\infty].</math> यह देखते हुए कि समुच्चय <math>A_i</math> जोड़ीवार असंयुक्त हैं,


:<math>B^{s,t}_k=\bigcup^m_{i=1}\Bigl(f^{-1}_k\Bigl([t\cdot c_i,+\infty]\Bigr)\cap A_i\Bigr).</math>
:<math>B^{s,t}_k=\bigcup^m_{i=1}\Bigl(f^{-1}_k\Bigl([t\cdot c_i,+\infty]\Bigr)\cap A_i\Bigr).</math>
पूर्व छवि के बाद से <math>f^{-1}_k\Bigl([t\cdot c_i,+\infty]\Bigr)</math> बोरेल सेट का
पूर्व छवि के पश्चात् से <math>f^{-1}_k\Bigl([t\cdot c_i,+\infty]\Bigr)</math> बोरेल समुच्चय का
<math>[t\cdot c_i,+\infty]</math> मापने योग्य फ़ंक्शन के अंतर्गत <math>f_k</math> मापने योग्य है, और <math>\sigma</math>-बीजगणित, परिभाषा के अनुसार, परिमित प्रतिच्छेदन और संघों के अंतर्गत बंद हैं, पहला प्रामाणित  इस प्रकार है।


चरण 3बी. दूसरे दावे को सिद्ध करना करने के लिए, प्रत्येक के लिए उस पर ध्यान दें <math>k</math> और हर <math>x\in X</math>, <math>f_k(x)\leq f_{k+1}(x).</math>
<math>[t\cdot c_i,+\infty]</math> मापने योग्य फलन के अंतर्गत <math>f_k</math> मापने योग्य है, और <math>\sigma</math>-बीजगणित, परिभाषा के अनुसार, परिमित प्रतिच्छेदन और संघों के अंतर्गत बंद हैं, पहला प्रामाणित इस प्रकार है।
चरण 3सी. तीसरे दावे को सिद्ध करना करने के लिए हम उसे दिखाते हैं <math>\textstyle X\subseteq\bigcup_k B^{s,t}_k</math>.
 
'''चरण 3बी.''' दूसरे दावे को सिद्ध करना करने के लिए, प्रत्येक के लिए उस पर ध्यान दें <math>k</math> और हर <math>x\in X</math>, <math>f_k(x)\leq f_{k+1}(x).</math>
 
'''चरण 3सी.''' तीसरे दावे को सिद्ध करना करने के लिए हम उसे दिखाते हैं <math>\textstyle X\subseteq\bigcup_k B^{s,t}_k</math>.


वास्तव में, यदि, इसके विपरीत, <math>\textstyle X\not\subseteq\bigcup_k B^{s,t}_k</math>, फिर तत्व
वास्तव में, यदि, इसके विपरीत, <math>\textstyle X\not\subseteq\bigcup_k B^{s,t}_k</math>, फिर तत्व
:<math>\textstyle x_0\in X\setminus\bigcup_k B^{s,t}_k=\bigcap_k(X\setminus B^{s,t}_k)</math>
:<math>\textstyle x_0\in X\setminus\bigcup_k B^{s,t}_k=\bigcap_k(X\setminus B^{s,t}_k)</math>
ऐसा उपस्तिथ है <math>f_k(x_0)<t\cdot s(x_0)</math>, हरएक के लिए <math>k</math>. सीमा मान कर <math>k\to\infty</math>, हम पाते हैं
ऐसा उपस्तिथ है <math>f_k(x_0)<t\cdot s(x_0)</math>, हर एक के लिए <math>k</math>. सीमा मान कर <math>k\to\infty</math>, हम पाते हैं
:<math>f(x_0)\leq t\cdot s(x_0)<s(x_0).</math>
:<math>f(x_0)\leq t\cdot s(x_0)<s(x_0).</math>
किन्तु प्रारंभिक धारणा से, <math>s\leq f</math>. यह विरोधाभास है.
किन्तु प्रारंभिक धारणा से, <math>s\leq f</math>. यह विरोधाभास है.


चरण 4. हर सरल के लिए <math>(\Sigma,\operatorname{\mathcal B}_{\R_{\geq 0}})</math>-मापने योग्य गैर-नकारात्मक कार्य <math>s_2</math>,
चरण 4. हर सरल के लिए <math>(\Sigma,\operatorname{\mathcal B}_{\R_{\geq 0}})</math>-मापने योग्य गैर-ऋणात्मक कार्य <math>s_2</math>,
:<math>\lim_n\int_{B^{s,t}_n}s_2\,d\mu=\int_Xs_2\,d\mu.</math>
:<math>\lim_n\int_{B^{s,t}_n}s_2\,d\mu=\int_Xs_2\,d\mu.</math>
इसे सिद्ध करने के लिए परिभाषित करें <math>\textstyle\nu(S)=\int_S s_2\,d\mu</math>. लेम्मा 1 द्वारा, <math>\nu(S)</math> पर उपाय है <math>\Omega</math>. नीचे से निरंतरता द्वारा (लेम्मा 2),
इसे सिद्ध करने के लिए परिभाषित करें <math>\textstyle\nu(S)=\int_S s_2\,d\mu</math>. लेम्मा 1 द्वारा, <math>\nu(S)</math> पर उपाय है <math>\Omega</math>. नीचे से निरंतरता द्वारा (लेम्मा 2),
Line 213: Line 216:
आवश्यकता अनुसार।
आवश्यकता अनुसार।


चरण 5. अब हम इसे प्रत्येक के लिए सिद्ध करते हैं <math>s\in\operatorname{SF}(f)</math>,
'''चरण 5.''' अभी हम इसे प्रत्येक के लिए सिद्ध करते हैं <math>s\in\operatorname{SF}(f)</math>,
:<math>\int_X s\,d\mu\leq\lim_k\int_X f_k\,d\mu.</math>
:<math>\int_X s\,d\mu\leq\lim_k\int_X f_k\,d\mu.</math>
मुख्य रूप से, की परिभाषा का उपयोग करते हुए <math>B^{s,t}_k</math>, की गैर-नकारात्मकता <math>f_k</math>, और लेबेस्ग इंटीग्रल की एकरसता (टिप्पणी 5 और रिमार्क 4 देखें), हमारे पास है
मुख्य रूप से, की परिभाषा का उपयोग करते हुए <math>B^{s,t}_k</math>, की गैर-ऋणात्मकता <math>f_k</math>, और लेबेस्ग इंटीग्रल की एकरसता (टिप्पणी 5 और रिमार्क 4 देखें), हमारे पास है
:<math>\int_{B^{s,t}_k}t\cdot s\,d\mu\leq\int_{B^{s,t}_k} f_k\,d\mu\leq\int_X f_k\,d\mu,</math>
:<math>\int_{B^{s,t}_k}t\cdot s\,d\mu\leq\int_{B^{s,t}_k} f_k\,d\mu\leq\int_X f_k\,d\mu,</math>
हरएक के लिए <math>k\geq 1</math>. चरण 4 के अनुसार, जैसे <math>k\to\infty</math>, असमानता हो जाती है
हरएक के लिए <math>k\geq 1</math>. चरण 4 के अनुसार, जैसे <math>k\to\infty</math>, असमानता हो जाती है
:<math>t\int_X s\,d\mu\leq\lim_k\int_X f_k\,d\mu.</math>
:<math>t\int_X s\,d\mu\leq\lim_k\int_X f_k\,d\mu.</math>
सीमा मान कर <math>t\uparrow 1</math> पैप्रामाणित
सीमा मान कर <math>t\uparrow 1</math> पैप्रामाणित र
:<math>\int_X s\,d\mu\leq\lim_k\int_X f_k\,d\mu,</math>
:<math>\int_X s\,d\mu\leq\lim_k\int_X f_k\,d\mu,</math>
आवश्यकता अनुसार।
आवश्यकता अनुसार।


चरण 6. अब हम विपरीत असमानता को सिद्ध करने में सक्षम हैं, अर्थात।
'''चरण 6.''' अभी हम विपरीत असमानता को सिद्ध करने में सक्षम हैं, अर्थात।
:<math> \int_X f \,d\mu \leq \lim_k \int_X f_k \,d\mu. </math>
:<math> \int_X f \,d\mu \leq \lim_k \int_X f_k \,d\mu. </math>
मुख्य रूप से, गैर-नकारात्मकता से, <math>f_+ = f</math> और <math>f_- = 0.</math> नीचे दी गई गणना के लिए, की गैर-नकारात्मकता <math>f</math> आवश्यक है। लेबेस्ग इंटीग्रल की परिभाषा और चरण 5 में स्थापित असमानता को प्रयुक्त करने पर, हमारे पास है
मुख्य रूप से, गैर-ऋणात्मकता से, <math>f_+ = f</math> और <math>f_- = 0.</math> नीचे दी गई गणना के लिए, की गैर-ऋणात्मकता <math>f</math> आवश्यक है। लेबेस्ग इंटीग्रल की परिभाषा और चरण 5 में स्थापित असमानता को प्रयुक्त करने पर, हमारे पास है
:<math> \int_X f \,d\mu=\sup_{s\in\operatorname{SF}(f)}\int_X s\,d\mu\leq\lim_k\int_X f_k\,d\mu.</math>
:<math> \int_X f \,d\mu=\sup_{s\in\operatorname{SF}(f)}\int_X s\,d\mu\leq\lim_k\int_X f_k\,d\mu.</math>
सबूत पूरा है.
प्रमाण पूरा है.


=='''यह भी देखें'''==
=='''यह भी देखें'''==
Line 236: Line 239:


{{reflist}}
{{reflist}}
[[Category: प्रमाण युक्त लेख]] [[Category: कलन में प्रमेय]] [[Category: अनुक्रम और श्रृंखला]] [[Category: वास्तविक विश्लेषण में प्रमेय]] [[Category: माप सिद्धांत में प्रमेय]]
[[it:Passaggio al limite sotto segno di integrale#Integrale di Lebesg


[[Category: Machine Translated Page]]
[[Category:Created On 25/07/2023]]
[[Category:Created On 25/07/2023]]
[[Category:Lua-based templates]]
[[Category:Machine Translated Page]]
[[Category:Pages with maths render errors]]
[[Category:Pages with script errors]]
[[Category:Short description with empty Wikidata description]]
[[Category:Templates Vigyan Ready]]
[[Category:Templates that add a tracking category]]
[[Category:Templates that generate short descriptions]]
[[Category:Templates using TemplateData]]
[[Category:अनुक्रम और श्रृंखला]]
[[Category:कलन में प्रमेय]]
[[Category:प्रमाण युक्त लेख]]
[[Category:माप सिद्धांत में प्रमेय]]
[[Category:वास्तविक विश्लेषण में प्रमेय]]

Latest revision as of 11:43, 10 August 2023

वास्तविक विश्लेषण के गणितीय क्षेत्र में, मोनोटोन अभिसरण प्रमेय अनेक संबंधित प्रमेयों में से एक है जो मोनोटोनिक अनुक्रमों (ऐसे अनुक्रम जो गैर-घटते या गैर-बढ़ते हैं) के अभिसरण (गणित) को सिद्ध करना करते हैं जो कि बंधा हुआ कार्य भी हैं। इस प्रकार अनौपचारिक रूप से, प्रमेय बताते हैं कि यदि कोई अनुक्रम बढ़ रहा है और ऊपर सर्वोच्च से घिरा हुआ है, तब अनुक्रम सर्वोच्च में परिवर्तित हो जाएगा; उसी तरह, यदि कोई अनुक्रम घट रहा है और नीचे एक अनंत से घिरा है, तब यह अनंत में परिवर्तित हो जाएगा।

वास्तविक संख्याओं के मोनोटोन अनुक्रम का अभिसरण

लेम्मा 1

यदि वास्तविक संख्याओं का अनुक्रम बढ़ रहा है और ऊपर से घिरा हुआ है, तब इसकी सर्वोच्च सीमा है।

प्रमाण

होने देना ऐसा क्रम हो, और चलो की शर्तों का समुच्चय हो . अनुमान से, गैर-रिक्त है और ऊपर से घिरा हुआ है। इस प्रकार वास्तविक संख्याओं की न्यूनतम-ऊपरी-सीमा वाली संपत्ति द्वारा, अस्तित्व में है और सीमित है। अभी, प्रत्येक के लिए , वहां उपस्तिथ ऐसा है कि , अन्यथा से की ऊपरी सीमा है , जो की परिभाषा के विपरीत है . तब से बढ़ रहा है, और प्रत्येक के लिए इसकी ऊपरी सीमा है , अपने पास है . इसलिए, परिभाषा के अनुसार, की सीमा है।

लेम्मा 2

यदि वास्तविक संख्याओं का कोई क्रम घट रहा हो और नीचे परिबद्ध हो, तब उसकी न्यूनतम सीमा होती है।

प्रमाण

प्रमाण उस मामले के प्रमाण के समान है जब अनुक्रम बढ़ रहा है और ऊपर से घिरा हुआ है।

प्रमेय

यदि वास्तविक संख्याओं का मोनोटोन अनुक्रम है (अर्थात्, यदि anan+1 प्रत्येक n ≥ 1 और anan+1 प्रत्येक n ≥ 1) तो इस अनुक्रम की एक सीमित सीमा होती है यदि और केवल यदि अनुक्रम परिबद्ध अनुक्रम है।[1]

प्रमाण

  • "यदि" -दिशा: प्रमाण सीधे लेम्मा से आता है।
  • "केवल यदि" -दिशा: (ε, δ) द्वारा - सीमा की परिभाषा, प्रत्येक अनुक्रम सीमित सीमा के साथ आवश्यक रूप से परिबद्ध है।

एक मोनोटोन श्रृंखला का अभिसरण

प्रमेय

यदि सभी प्राकृतिक संख्याओं j और k के लिए, k, aj,k गैर-ऋणात्मक वास्तविक संख्या है और aj,kaj+1,k, तो[2]: 168 

प्रमेय कहता है कि यदि आपके पास गैर-ऋणात्मक वास्तविक संख्याओं का अनंत आव्युह है

  1. कॉलम अशक्त रूप से बढ़ रहे हैं और बंधे हुए हैं, और
  2. प्रत्येक पंक्ति के लिए, श्रृंखला (गणित) जिसके पद इस पंक्ति द्वारा दिए गए हैं, उसका एक अभिसरण योग है,

तब पंक्तियों के योग की सीमा उस श्रृंखला के योग के सामान्तर होती है जिसका पद k स्तंभ k की सीमा द्वारा दिया जाता है (जो इसका सर्वोच्च भी है)। श्रृंखला में अभिसरण योग होता है यदि और केवल यदि पंक्ति योगों का (अशक्त रूप से बढ़ता हुआ) क्रम परिबद्ध है और इसलिए अभिसरण है।

उदाहरण के तौर पर, पंक्तियों की अनंत श्रृंखला पर विचार करें

जहां n अनंत तक पहुंचता है (इस श्रृंखला की सीमा e (गणितीय स्थिरांक) है)। यहां पंक्ति n और कॉलम k में आव्युह प्रविष्टि है

कॉलम (निश्चित k) वास्तव में n के साथ अशक्त रूप से बढ़ रहे हैं और (1/k से!) बंधे हुए हैं, जबकि पंक्तियों में केवल सीमित रूप से अनेक गैर-शून्य पद हैं, इसलिए शर्त 2 संतुष्ट है; प्रमेय अभी कहता है कि आप पंक्ति योग की सीमा की गणना कर सकते हैं अर्थात्, स्तंभ सीमाओं का योग लेकर.

बेप्पो लेवी की लेम्मा

निम्नलिखित परिणाम बेप्पो लेवी के कारण है, जिन्होंने सत्र 1906 में हेनरी लेब्सग्यू द्वारा पहले के परिणाम का थोड़ा सा सामान्यीकरण सिद्ध करना किया था।[3] इस प्रकार जो आगे हुआ, को दर्शाता है - बोरेल का बीजगणित चालू होता है . परिभाषा से, समुच्चय सम्मिलित है और सभी बोरेल उपसमुच्चय

प्रमेय

होने देना माप हो (गणित), और . बिंदुवार गैर-घटते क्रम पर विचार करें का -मापने योग्य कार्य गैर-ऋणात्मक कार्य , अर्थात, प्रत्येक के लिए और हर ,

अनुक्रम की बिन्दुवार सीमा निर्धारित करें होना . अर्थात हर किसी के लिए ,

तब है -मापने योग्य और

टिप्पणी 1. अभिन्न अंग परिमित या अनंत हो सकते हैं।

टिप्पणी 2. यदि इसकी धारणाएँ मान्य हैं तब प्रमेय सत्य रहता है -लगभग हर स्थान। दूसरे शब्दों में, यह पर्याप्त है कि शून्य समुच्चय है ऐसा कि क्रम प्रत्येक के लिए गैर-कमी यह देखने के लिए कि यह सच क्यों है, हम अवलोकन से प्रारंभ करते हैं जो अनुक्रम की अनुमति देता है बिंदुवार गैर-घटाना लगभग हर स्थान इसकी बिंदुवार सीमा का कारण बनता है कुछ शून्य समुच्चय पर अपरिभाषित होना . उस शून्य समुच्चय पर, फिर इच्छानुसार से परिभाषित किया जा सकता है, उदाहरण के लिए शून्य के रूप में, या किसी अन्य तरीके से जो मापनीयता को सुरक्षित रखता है। यह देखने के लिए कि यह प्रमेय के परिणाम को प्रभावित क्यों नहीं करेगा, तब से ध्यान दें हमारे पास, हर किसी के लिए है

और

उसे उपलब्ध कराया है -मापने योग्य.[4]: धारा 21.38  (यह समानताएं गैर-ऋणात्मक फलन के लिए लेबेस्ग इंटीग्रल की परिभाषा से सीधे अनुसरण करती हैं)।

टिप्पणी 3. प्रमेय की मान्यताओं के अनुसार ,

(ध्यान दें कि समानता की दूसरी श्रृंखला टिप्पणी 5 से अनुसरण करती है)।

टिप्पणी 4. नीचे दिया गया प्रमाण यहां स्थापित किए गए को छोड़कर लेबेस्ग इंटीग्रल के किसी भी गुण का उपयोग नहीं करता है। इस प्रकार, प्रमेय का उपयोग लेबेस्ग एकीकरण से संबंधित अन्य बुनियादी गुणों, जैसे कि रैखिकता, को सिद्ध करना करने के लिए किया जा सकता है।

टिप्पणी 5 (लेबेस्ग इंटीग्रल की एकरसता)। नीचे दिए गए प्रमाण में, हम लेबेस्ग इंटीग्रल के मोनोटोनिक गुण को केवल गैर-ऋणात्मक कार्यों पर प्रयुक्त करते हैं। विशेष रूप से (टिप्पणी 4 देखें), कार्य करें होना -मापने योग्य.

  • यदि हर स्थान पर तब
  • यदि और तब

प्रमाण। निरूपित सरल का समुच्चय -मापने योग्य कार्य ऐसा है कि हर स्थान पर 1. चूँकि अपने पास

लेबेस्ग इंटीग्रल की परिभाषा और सुप्रीमम के गुणों के अनुसार,

2. चलो समुच्चय का सूचक कार्य हो इसका अनुमान लेबेस्ग इंटीग्रल की परिभाषा से लगाया जा सकता है

यदि हम उस पर ध्यान दें, प्रत्येक के लिए के बाहर पिछली संपत्ति के साथ संयुक्त, असमानता तात्पर्य

प्रमाण

यह प्रमाण फ़तौ की प्रमेयिका पर निर्भर नहीं करता है; चूँकि, हम बताते हैं कि उस लेम्मा का उपयोग कैसे किया जा सकता है। जो लोग प्रमाण की इस स्वतंत्रता में रुचि नहीं रखते हैं वह नीचे दिए गए मध्यवर्ती परिणामों को छोड़ सकते हैं।

मध्यवर्ती परिणाम

लेब्सग्यू माप के रूप में अभिन्न

लेम्मा 1. चलो मापने योग्य स्थान बनें. सरल पर विचार करें -मापने योग्य गैर-ऋणात्मक कार्य . उपसमुच्चय के लिए , परिभाषित करना

तब पर उपाय है .

प्रमाण

एकरसता टिप्पणी 5 से आती है। यहां, हम केवल गणनीय योगात्मकता सिद्ध करेंगे, बाकी पाठक पर छोड़ देंगे। होने देना , जहां सभी समुच्चय जोड़ीवार असंयुक्त हैं. सरलता के कारण,

कुछ परिमित गैर-ऋणात्मक स्थिरांकों के लिए और जोड़ीवार असंयुक्त समुच्चय ऐसा है कि . लेब्सेग इंटीग्रल की परिभाषा के अनुसार,

चूंकि सभी समुच्चय जोड़ीवार असंयुक्त हैं, गणनीय योगात्मकता

हमें देता है

चूँकि सभी सारांश गैर-ऋणात्मक हैं, श्रृंखला का योग, चाहे यह योग परिमित हो या अनंत, यदि योग क्रम बदलता है तब नहीं बदल सकता। इसी कारणवश,

आवश्यकता अनुसार।

"नीचे से निरंतरता"

निम्नलिखित संपत्ति माप की परिभाषा का प्रत्यक्ष परिणाम है।

लेम्मा 2. चलो उपाय हो, और , कहाँ

अपने सभी समुच्चयों के साथ गैर-घटती हुई श्रृंखला है -मापने योग्य. तब

प्रमेय का प्रमाण

चरण 1. हम इसे दिखाकर शुरुआत करते हैं है -मापने योग्य.[4]: धारा 21.3 

टिप्पणी। यदि हम फ़तौ की लेम्मा का उपयोग कर रहे थे, तब मापनीयता टिप्पणी 3(ए) से आसानी से अनुसरण करेगी।

फ़तौ के लेम्मा का उपयोग किए बिना ऐसा करने के लिए, यह दिखाना पर्याप्त है कि अंतराल की उलटी छवि अंतर्गत सिग्मा-बीजगणित का तत्व है पर , क्योंकि (बंद) अंतराल वास्तविक पर बोरेल सिग्मा बीजगणित उत्पन्न करते हैं। तब से बंद अंतराल है, और, प्रत्येक के लिए , ,

इस प्रकार,

एक के नीचे स्थापित बोरेल की उलटी छवि होना -मापने योग्य कार्य , गणनीय प्रतिच्छेदन में प्रत्येक समुच्चय का तत्व है . तब से -बीजगणित, परिभाषा के अनुसार, गणनीय प्रतिच्छेदन के अंतर्गत बंद होते हैं, इससे पता चलता है है -मापने योग्य, और अभिन्न अच्छी तरह से परिभाषित है (और संभवतः अनंत)।

चरण 2. हम सबसे पहले वो दिखाएंगे की परिभाषा और की एकरसता इसका कारणयह है , हरएक के लिए और हर . लेबेस्ग इंटीग्रल की एकरसता (या, अधिक त्रुटिहीन रूप से, रिमार्क 5 में स्थापित इसका संकीर्ण संस्करण; रिमार्क 4 भी देखें) द्वारा,

और

ध्यान दें कि दाईं ओर की सीमा उपस्तिथ है (सीमित या अनंत) क्योंकि, एकरसता के कारण (टिप्पणी 5 और टिप्पणी 4 देखें), अनुक्रम गैर-घटता नहीं है।

चरण 2 का अंत.

अभी हम विपरीत असमानता को सिद्ध करते हैं। हम यह दिखाना चाहते हैं

.

फ़तौ की लेम्मा का उपयोग करके प्रमाण। टिप्पणी 3 के अनुसार, जिस असमानता को हम सिद्ध करना चाहते हैं वह इसके समतुल्य है

किन्तु पश्चात् वाला फ़तौ की लेम्मा से तुरंत अनुसरण करता है, और प्रमाण पूरा हो गया है।

स्वतंत्र प्रमाण. फ़तौ की लेम्मा का उपयोग किए बिना असमानता को सिद्ध करना करने के लिए, हमें कुछ अतिरिक्त मशीनरी की आवश्यकता है। निरूपित सरल का समुच्चय -मापने योग्य कार्य ऐसा है कि पर .

चरण 3. सरल कार्य दिया गया है और वास्तविक संख्या , परिभाषित करना

तब , , और .

चरण 3ए. पहले दावे को सिद्ध करना करने के लिए आइए , जोड़ीवार असंयुक्त मापन योग्य समुच्चयों के कुछ सीमित संग्रह के लिए ऐसा है कि , कुछ (परिमित) गैर-ऋणात्मक स्थिरांक , और समुच्चय के सूचक फलन को दर्शाते हुए .

हरएक के लिए यदि और केवल यदि धारण करता है यह देखते हुए कि समुच्चय जोड़ीवार असंयुक्त हैं,

पूर्व छवि के पश्चात् से बोरेल समुच्चय का

मापने योग्य फलन के अंतर्गत मापने योग्य है, और -बीजगणित, परिभाषा के अनुसार, परिमित प्रतिच्छेदन और संघों के अंतर्गत बंद हैं, पहला प्रामाणित इस प्रकार है।

चरण 3बी. दूसरे दावे को सिद्ध करना करने के लिए, प्रत्येक के लिए उस पर ध्यान दें और हर ,

चरण 3सी. तीसरे दावे को सिद्ध करना करने के लिए हम उसे दिखाते हैं .

वास्तव में, यदि, इसके विपरीत, , फिर तत्व

ऐसा उपस्तिथ है , हर एक के लिए . सीमा मान कर , हम पाते हैं

किन्तु प्रारंभिक धारणा से, . यह विरोधाभास है.

चरण 4. हर सरल के लिए -मापने योग्य गैर-ऋणात्मक कार्य ,

इसे सिद्ध करने के लिए परिभाषित करें . लेम्मा 1 द्वारा, पर उपाय है . नीचे से निरंतरता द्वारा (लेम्मा 2),

आवश्यकता अनुसार।

चरण 5. अभी हम इसे प्रत्येक के लिए सिद्ध करते हैं ,

मुख्य रूप से, की परिभाषा का उपयोग करते हुए , की गैर-ऋणात्मकता , और लेबेस्ग इंटीग्रल की एकरसता (टिप्पणी 5 और रिमार्क 4 देखें), हमारे पास है

हरएक के लिए . चरण 4 के अनुसार, जैसे , असमानता हो जाती है

सीमा मान कर पैप्रामाणित र

आवश्यकता अनुसार।

चरण 6. अभी हम विपरीत असमानता को सिद्ध करने में सक्षम हैं, अर्थात।

मुख्य रूप से, गैर-ऋणात्मकता से, और नीचे दी गई गणना के लिए, की गैर-ऋणात्मकता आवश्यक है। लेबेस्ग इंटीग्रल की परिभाषा और चरण 5 में स्थापित असमानता को प्रयुक्त करने पर, हमारे पास है

प्रमाण पूरा है.

यह भी देखें

टिप्पणियाँ

  1. A generalisation of this theorem was given by Bibby, John (1974). "Axiomatisations of the average and a further generalisation of monotonic sequences". Glasgow Mathematical Journal. 15 (1): 63–65. doi:10.1017/S0017089500002135.
  2. See for instance Yeh, J. (2006). Real Analysis: Theory of Measure and Integration. Hackensack, NJ: World Scientific. ISBN 981-256-653-8.
  3. Schappacher, Norbert; Schoof, René (1996), "Beppo Levi and the arithmetic of elliptic curves" (PDF), The Mathematical Intelligencer, 18 (1): 60, doi:10.1007/bf03024818, MR 1381581, S2CID 125072148, Zbl 0849.01036
  4. 4.0 4.1 See for instance Schechter, Erik (1997). Handbook of Analysis and Its Foundations. San Diego: Academic Press. ISBN 0-12-622760-8.